LSAT and Law School Admissions Forum

Get expert LSAT preparation and law school admissions advice from PowerScore Test Preparation.

User avatar
 Dave Killoran
PowerScore Staff
  • PowerScore Staff
  • Posts: 5853
  • Joined: Mar 25, 2011
|
#71215
This game is discussed in our Podcast: LSAT Podcast Episode 31: The September 2019 LSAT Logic Games Section


Complete Question Explanation
(The complete setup for this game can be found here: https://forum.powerscore.com/lsat/viewtopic.php?t=31306)

The correct answer choice is (D).

If you made a hypothetical for the question #9, then this is an easy one. As our hypothetical there shows, P can indeed be in case 1, and thus answer choice (D) is correct:

  • J/P ..... G/N ..... _S_ ..... N/G ..... P/J ..... _M_ ..... _H_
     1 .....          2 .....    3 .....       4 .....      5 .....      6 .....      7

Answer choice (A): This scenario does not allow for sufficient room for J, P, and S to be separated.

Answer choice (B): This scenario does not allow for sufficient room for J, P, and S to be separated.

Answer choice (C): This scenario does not allow room for the MH block.

Answer choice (D): This is the correct answer choice.

Answer choice (E): This scenario does not allow room for the MH block.

Get the most out of your LSAT Prep Plus subscription.

Analyze and track your performance with our Testing and Analytics Package.